Đến nội dung

LoveMath1234567

LoveMath1234567

Đăng ký: 13-12-2018
Offline Đăng nhập: 11-04-2019 - 06:35
-----

Trong chủ đề: Phương pháp giải Phương trình

21-01-2019 - 15:43

Phương pháp cô lập tham số.

pp bạn nói đa phần để xét tính đơn điệu của hàm số

Cho mình nói thêm :

 Sau khi đặt ẩn t thầy m vẽ bảng biến thiên của hàm số : y= t bình -4t

Để pt có 4 nghiệm thì m phải lớn hơn -4 nhỏ hơn -3 .....

Bạn có thể xem cho mình xem đây là pp j ko ...Mình cần hk nó gấp


Trong chủ đề: Cho $A(1;2)$, $B(3;4)$, tìm trên $Ox$ điểm...

18-01-2019 - 06:34

a) Lấy A' đối xứng với A qua trục hoành. Đặt N là giao điểm của A'B với Ox

Với mọi M thuộc Ox ta có

MA + MB = MA' +MB $\geq A'B=\sqrt{(3-1)^{2}+(4+2)^{2}}=2\sqrt{10}$

Đẳng thức xảy ra <=> M$\equiv$N<=> M thuộc đoạn HK

và $\frac{MH}{MK}=\frac{A'H}{BK}=\frac{2}{4}=\frac{1}{2}$

<=> $\frac{\overline{MH}}{\overline{MK}}= -\frac{1}{2}$

<=> $x_{M}=\frac{x_{H}+\frac{1}{2}x_{K}}{1+\frac{1}{2}}=\frac{1+\frac{3}{2}}{\frac{3}{2}}=\frac{5}{3}$

<=> M(5/3;0)

HK ở đâu ra thế ạ


Trong chủ đề: Trong các tập hợp sau, tập nào là tập con của tập nào ?

06-01-2019 - 21:12

 

a) DBCADBCA

 

b) AB

 

sao b lại là con c thế bạn


Trong chủ đề: Trong các tập hợp sau, tập nào là tập con của tập nào ?

06-01-2019 - 21:08

a) $D\subset B\subset C\subset A$

b) $A\subset B$

thế nghĩa là hình chữ nhật là  hình thoi????.....


Trong chủ đề: Tổng hợp các bài BĐT

02-01-2019 - 21:26

 

Topic này chủ yếu là coppy nội dung đề bài và lời giải của các mem lại, các ĐHV THPT có thể coppy tiếp CD13 nhưng nên để chung trong một khung. Tính đúng sai của các lời giải thì kiểm tra lại sau.

 

Bắt đầu vậy!

 

Bài 1:

Cho abc=1 va $a^{3}> 36.  CMR  :\frac{a^{2}}{3}+b^{2}+c^{2}> ab +bc+ca$}

Lời giải:

$VT-VP=\frac{a^{2}}{4}+b^{2}+c^{2}-ab-bc+2bc+\frac{a^{2}}{12}=(\frac{a}{2}-b-c)^{2}+\frac{a^{2}-36bc}{12}>0\Rightarrow$ đpcm

Cách khác:

Từ giả thiết suy ra $a>0$ và $bc>0$. Bất đẳng thức cần chứng minh tương đương với
\[\dfrac{a^2}{3}+(b+c)^2-3bc-a(b+c)\ge 0\\ \iff \dfrac{1}{3}+\left(\dfrac{b+c}{a}\right)^2-\dfrac{b+c}{a}-\dfrac{3}{a^3}\ge 0\]
Vì $a^3>36$ nên \[\dfrac{1}{3}+\left(\dfrac{b+c}{a}\right)^2-\dfrac{b+c}{a}-\dfrac{3}{a^3}> \left(\dfrac{b+c}{a}\right)^2-\dfrac{b+c}{a}+ \dfrac{1}{4}= \left(\dfrac{b+c}{a}-\dfrac{1}{2}\right)^2 >0\]
 
 

 

Bài 2:

Với a,b,c >0; n ∈ N*.CMR:

$\frac{a^{n}}{b+c}+\frac{b^{n}}{a+c}+\frac{c^{n}}{a+b}\geq \frac{3}{2}\left ( \frac{a^{n}+b^{n}+c^{n}}{a+b+c} \right )$

Lời giải:

$\sum \frac{a^{n}}{b+c}\geq \frac{1}{3}(\sum a^{n})(\sum \frac{1}{a+b})\geq \frac{1}{3}(\sum a^{n})(\frac{9}{2(a+b+c)})=\frac{3}{2}(\frac{\sum a^{n}}{\sum a})$

 

 

 

Bài 3:

Cho $x,y,z >0$ thỏa điều kiện $x^{2}+y^{2}+z^{2}=9$

Tìm giá trị nhỏ nhất của $P=\frac{x^5}{y^2}+\frac{y^5}{z^2}+\frac{z^5}{x^2}$

Lời giải:

Theo $Cauchy$ Ta có:

$$\dfrac{x^5}{y^2}+\dfrac{x^5}{y^2}+\sqrt{3}y^2+\sqrt{3}y^2+3\sqrt{3}\ge \sqrt{3}x^2$$

Cách khác:

Sử dụng Cauchy-Schwarzt ta có 

       $\frac{x^5}{y^2}+\frac{y^5}{z^2}+\frac{z^5}{x^2}\geqslant \frac{(x^3+y^3+z^3)^2}{xy^2+yz^2+zx^2}$

Sử dụng Cauchy-Schwarzt và AM-GM ta có 

       $xy^2+yz^2+zx^2\leqslant \sqrt{(x^2+y^2+z^2)(x^2y^2+y^2z^2+z^2x^2)}\leqslant \sqrt{\frac{(x^2+y^2+z^2)^3}{3}}=3$

Do đó $P\geqslant \frac{(x^3+y^3+z^3)^2}{3}\geqslant \frac{(x^2+y^2+z^2)^3}{9}=3$ 

Đẳng thức xảy ra khi $x=y=z=1$

 

 

 

Bài 4:

Ch0 $a>0$ và $n$ là 1 số tự nhiên

Chứng minh rằng $a^n+\frac{1}{a^n}-2\geqslant n^2(a+\frac{1}{a}-2)$

Lời giải:

Bất đẳng thức tương đương với $(a^{n-1}+a^{n-2}+...+a+1)\geq n^2a^{n-1}$ (hiển nhiên theo AM-GM)

Cách khác:

Do tính đối xứng giữa a và $\frac{1}{a}$ nên ta có thể giả sử a ≥ 1.  đặt $\sqrt{a}$ =x ≥ 1.bdt $\Leftrightarrow$ $x^{2n}+\frac{1}{x^{2n}}-2 \geq n^{2}(x^{2}+\frac{1}{x^{2}}-2)\Leftrightarrow (x^{n}-\frac{1}{x^{n}})^{2}\geq n^{2}(x-\frac{1}{x})^{2} \Leftrightarrow $x^{n}-\frac{1}{x^{n}}\geq n(x-\frac{1}{x})$①.

Với x=1 thì ① đúng

Với x>1 thì ① $\Leftrightarrow x^{n-1} +x^{n-3} ...+\frac{1}{x^{n-3}}+\frac{1}{x^{n-1}}\geq n$ (đúng vì theo bđt AM-GM).

Dấu bằng xảy ra khi x=1 $\Leftrightarrow a=1$

 

 

 

Bài 5:

Cho $a,b,c,d$ là các số thực thỏa mãn $\left\{\begin{matrix} a+b+c+d=0\\a^2+b^2+c^2+d^2=2 \end{matrix}\right.$

Tìm GTLN của $P=abcd$

Lời giải:

Áp dụng AM-GM ta có 

$2=\sum a^{2}\geq 4\sqrt[4]{\prod a^{2}}\Rightarrow \sqrt{\left | abcd \right |}\leq \frac{1}{2}\Rightarrow abcd\leq \frac{1}{4}$

Dấu bằng xảy ra khi $a=b=-c=-d=\frac{1}{\sqrt{2}}$ và các hoán vị của chúng

 

 

 

Bài 6:

Cho $a,\,b,\,c\geq 0$ thỏa mãn $a+b+c=1.$ Tìm giá trị lớn nhất của biểu thức: $$P=abc\left(a^2+b^2+c^2\right)$$

Lời giải:

Ta có: $P=abc(a+b+c)(a^{2}+b^{2}+c^{2})\leq \frac{1}{3}(ab+bc+ca)^{2}(a^{2}+b^{2}+c^{2})$

 

Mặt khác, lại có: $(ab+bc+ca)^{2}(a^{2}+b^{2}+c^{2})\leq \left (\frac{(a+b+c)^{2}}{3} \right )^{3}=\frac{1}{27}$

 

Do đó: $P\leq \frac{1}{81}$

 

Dấu bằng xảy ra khi và chỉ khi $a=b=c= \frac{1}{3}$

 

 

 

Bài 7:

Cho các số thực $x,\,y>0$ thỏa mãn $3x+y\leq1.$ Tìm giá trị nhỏ nhất của biểu thức: $$S=\dfrac{1}{x}+\dfrac{1}{\sqrt{xy}}$$

Lời giải:

$S\geq \frac{1}{x}+\frac{1}{\sqrt{x(1-3x)}}$

$\geq \frac{1}{x}+\frac{2}{1-2x}=\frac{2}{x(1-x)}\geq \frac{8}{(x+1-x)^{2}}=8$

Dấu "=" xảy ra $\Leftrightarrow x=\frac{1}{4}$

 

 

 

Bài 8:

Cho các số thực a,b,c,x,y thỏa mãn $$ax-by=\sqrt{3}$$ .

Tìm GTNN của $F= a^{2}+b^{2}+x^{2}+y^{2}+ bx +ay$

Lời giải:

Sử dụng giả thiết $ax-by=\sqrt{3}$ ta có:
$$(a^2+b^2)(x^2+y^2)=(ax+by)^2+(ax-by)^2=(ax+by)^2+3$$
Áp dụng bất đẳng thức $Cauchy$ , suy ra:
$$a^2+b^2=x^2+y^2=(a^2+b^2)+(x^2+y^2) \\ \ge 2\sqrt{(a^2+b^2)(x^2+y^2)}=2\sqrt{(ax+by)^2+3}$$
Do đó, ta đưa về bài toán tìm GTNN của: $2\sqrt{x^2+3}+x$ trong đó $x=ax+by$
Ta có:
$$\left(2\sqrt{x^2+3}+x\right)^2=4(x^2+3)+4x\sqrt{x^2+3}+x^2 \\ = (x^2+3)+4x\sqrt{x^2+3}+4x^2+9 \\ = \left(\sqrt{x^2+3}+2x\right)^2+9\ge 9$$
$$\Rightarrow 2\sqrt{x^2+3}+x\ge 3$$
Vậy $\text{MinT}=\fbox{3}$
 
 
 

Bài 9:

Cho các số thực dương $a,b,c$. Tìm giá trị nhỏ nhất của biểu thức:

 

                                $P=\frac{2}{a+ \sqrt{ab}+ \sqrt[3]{abc}}-\frac{3}{\sqrt{a+b+c}}$

Lời giải:

$a+\sqrt{\frac{1}{2}a.2b}+\sqrt[3]{\frac{1}{4}a.b.4c}\leq a+\frac{1}{4}a+b+\frac{1}{12}a+\frac{1}{3}b+\frac{4}{3}c=\frac{4}{3}(a+b+c)$

Do đó $P\geq \frac{3}{2(a+b+c)}-\frac{3}{\sqrt{a+b+c}}$...

 

 

 

Bài 10:

Cho x,y là các số không âm thoả $x^{3}+y^{3}\leq 1$

 

Tìm giá trị lớn nhất của $P=2\sqrt{x}+\sqrt{y}$

Lời giải:

$(x^3+y^3)(\sqrt[5]{2^6}+1)^5\geqslant (2\sqrt{x}+\sqrt{y})^6$

$\Leftrightarrow 2\sqrt{x}+\sqrt{y}\leqslant \sqrt[6]{(\sqrt[5]{2^6}+1)^5}$

Vậy $Max(P)= \sqrt[6]{(\sqrt[5]{2^6}+1)^5}\Leftrightarrow \frac{a^3}{2\sqrt[5]{2}}=b^3=\frac{1}{2\sqrt[5]{2}+1}$

 

Chiều coppy tiếp, sau đó kiểm tra nội dung sau.

 

 

Bài 11:Cho các số a,b,c không âm không đồng thời bằng không. Chứng minh rằng;

$\sum \frac{2a^{2}-bc}{b^{2}-bc+c^{2}}\geq 3$

 

Lời giải:(vutuanhien)

 

Không mất tính tổng quát, ta có thể giả sử $b$ là số nằm giữa $a$ và $c$

BĐT đã cho tương đương với

$$\sum \frac{2a^2+(b-c)^2}{b^2-bc+c^2}\geq 6$$

Áp dụng BĐT Cauchy-Schwarz, ta có

$$\sum \frac{2a^2}{b^2-bc+c^2}\geq \frac{2(a^2+b^2+c^2)^2}{\sum a^2(b^2-bc+c^2)}=\frac{2(a^2+b^2+c^2)^2}{2\sum a^2b^2-abc\sum a}$$

$$\sum \frac{(b-c)^2}{b^2-bc+c^2}\geq \frac{[a(b-c)+b(a-c)+c(a-b)]^2}{2\sum a^2b^2-abc\sum a}=\frac{4b^2(a-c)^2}{2\sum a^2b^2-abc\sum a}$$

Do đó ta chỉ cần chứng minh

$$(a^2+b^2+c^2)^2+2b^2(a-c)^2\geq 6\sum a^2b^2-3abc\sum a (1)$$

Ta có 

$b^2(a-c)^2=[a(b-c)+c(a-b)]^2=a^2(b-c)^2+c^2(a-b)^2+2ac(a-b)(b-c)$

$\geq a^2(b-c)^2+c^2(a-b)^2$

Suy ra 

$$2b^2(a-c)^2\geq a^2(b-c)^2+b^2(c-a)^2+c^2(a-b)^2$$

$$\Rightarrow VT (1)\geq (\sum a^2)^2+2\sum a^2b^2-2abc\sum a$$

Do đó ta chỉ còn phải chứng minh 

$$(\sum a^2)^2+2\sum a^2b^2-2abc\sum a\geq 6\sum a^2b^2-3abc\sum a$$

$$\Leftrightarrow \sum a^4+abc\sum a\geq 2\sum a^2b^2$$

BĐT này hiển nhiên đúng theo BĐT Schur

$$\sum a^4+abc\sum a\geq \sum ab(a^2+b^2)$$

Và BĐT AM-GM

$$\sum ab(a^2+b^2)\geq 2\sum a^2b^2$$

Kết thúc chứng minh 

Đẳng thức xảy ra khi $a=b=c$ hoặc $a=b$, $c=0$ và các hoán vị.

 

Bài 12:(bosulan239)

Cho a,b,c là các số không âm không đồng thời bằng không.

CMR

$\frac{\sum a^{2}}{\sum ab}\geq \sum \frac{ab}{b^{2}+bc+c^{2}}$

Bài giải:(vutuanhien)

 

BĐT đã cho tương đương với

$\frac{a^2}{ab+bc+ca}-\frac{ab}{b^2+bc+c^2}+\frac{b^2}{ab+bc+ca}-\frac{bc}{c^2+ca+a^2}+\frac{c^2}{ab+bc+ca}-\frac{ca}{a^2+ab+b^2}\geq 0$

$\Leftrightarrow \sum \frac{ac(ac-b^2)}{b^2+bc+c^2}\geq 0$

Do $\frac{ac(ac-b^2)}{b^2+bc+c^2}=\frac{ac^2(a+b+c)}{b^2+bc+c^2}-ac$ nên BĐT đã cho có thể viết lại thành

$\sum \frac{ac^2(a+b+c)}{b^2+bc+c^2}\geq ab+bc+ca$

$\Leftrightarrow \sum \frac{ac^2}{b^2+bc+c^2}\geq \frac{ab+bc+ca}{a+b+c}$

Áp dụng BĐT Cauchy-Schwarz, ta có

$VT\geq \frac{(ab+bc+ca)^2}{\sum a(b^2+bc+c^2)}=\frac{ab+bc+ca}{a+b+c}$

Kết thúc chứng minh 

 

 


Bài 13: (nguyencuong123)

Cho a,b,c không âm thoả mãn: $a+b+c=3$

Chứng Minh: $\sum \frac{a+1}{ab+1}\geq 3$

 

 

Bài giải:(Juliel)

Áp dụng AM-GM cho vế trái, ta cần chứng minh :

$(a+1)(b+1)(c+1)\geq (ab+1)(bc+1)(ca+1)\Leftrightarrow abc+(ab+bc+ca)+(a+b+c)+1\geq a^{2}b^{2}c^{2}+abc(a+b+c)+(ab+bc+ca)+1\Leftrightarrow abc+4\geq a^{2}b^{2}c^{2}+3abc+1\Leftrightarrow a^{2}b^{2}c^{2}+2abc\leq 3$

Hiển nhiên đúng vì $abc\leq (\frac{a+b+c}{3})^{3}=1$

 


 


Bài 14:(Chrome98):Chứng minh bất đẳng thức sau với $a,b,c>0$ và $a+b+c=1$:

 

\[ \frac{a^2}{3a+1}+\frac{b^2}{3b+1}+\frac{c^2}{3c+1}\ge 24\left(\frac{a^2}{9a+1}+\frac{b^2}{9b+1}+\frac{c^2}{9c+1}\right)^2 \]

Bài giải: (Simpson Joe Donald)

 

 
 
$\bullet\ AM-GM:\ \dfrac{a^2}{9a+1}= \dfrac{a^2}{6a+(3a+1)}\le \dfrac{a^2}{2\sqrt{6a(3a+1)}}= \dfrac{a\sqrt{a}}{2\sqrt{6(3a+1)}}$ ;
$\bullet\ Cauchy-Schwarz:\ VP\le \left( \dfrac{a\sqrt{a}}{\sqrt{3a+1}}+  \dfrac{b\sqrt{b}}{\sqrt{3b+1}}+  \dfrac{c\sqrt{c}}{\sqrt{3c+1}}\right)^2\le (a+b+c).VT=VT$

 

Bài 15:(trauvang97:)Cho các số thực dương $a,b,c$ thoả mãn:

 

                             $\frac{2}{a^{2}+1}+\frac{2}{b^{2}+1}+\frac{2}{c^{2}+1}\geq 3$

 

Chứng minh rằng: $(a-2)^{2}+(b-2)^{2}+(c-2)^{2}\geq 3$

 

Bài giải:

 

(Nguyen Huy Tuyen)$\frac{2}{a^{2}+1}+\frac{2}{b^{2}+1}+\frac{2}{c^{2}+1}\geq 3\Leftrightarrow \sum \frac{(1-a)(1+a)}{a^2+1}\geqslant 0$

$(a-2)^{2}+(b-2)^{2}+(c-2)^{2}-3=\sum (a-3)(a-1)$

Ta có :$\sum (a-3)(a-1)-\sum \frac{2(1-a)(1+a)}{a^2+1}=\sum \frac{(a-1)^4}{a^2+1}\geqslant 0$

           $\Leftrightarrow \sum (a-3)(a-1)\geqslant \sum \frac{2(1-a)(1+a)}{a^2+1}\geqslant 0$

           $\Leftrightarrow (a-2)^{2}+(b-2)^{2}+(c-2)^{2}\geq 3$

 

 

Bài 16:(phanquockhanh)Cho $x,y,z >0 : xyz+x+z=y$ . Tìm giá trị lớn nhất của biểu thức : $P=\frac{2}{x^2+1} - \frac{2}{y^2+1} -\frac{4z}{\sqrt{z^2+1}}+\frac{3z}{(z^2+1).\sqrt{z^2+1}}$

(Trích đề thi thử số 2 – THTT)

Bài giải:

(trauvang97)Từ giả thiết ta có: $x=\frac{y-z}{1+yz}$.

 

Khi đó:

 

$P=\frac{2(1+yz)^{2}}{(y^{2}+1)(z^{2}+1)}-\frac{2}{y^{2}+1}-\frac{4z}{\sqrt{z^{2}+1}}+\frac{3z}{(z^{2}+1)\sqrt{z^{2}+1}}$

 

$P=\frac{2z(2y+(y^{2}-1)z)}{(y^{2}+1)(z^{2}+1)}-\frac{4z}{\sqrt{z^{2}+1}}+\frac{3z}{(z^{2}+1)\sqrt{z^{2}+1}}$

 

Do $\frac{2z(2y+(y^{2}-1)z)}{(y^{2}+1)(z^{2}+1)}=\frac{2z\sqrt{(2y+(y^{2}-1)z)^{2}}}{(y^{2}+1)(z^{2}+1)}\leq \frac{2z\sqrt{(4y^{2}+(y^{2}-1)^{2})(1+z^{2})}}{(y^{2}+1)(z^{2}+1)}=\frac{2z}{\sqrt{z^{2}+1}}$

 

Do đó:

 

$P\leq \frac{2z}{\sqrt{z^{2}+1}}-\frac{4z}{\sqrt{z^{2}+1}}+\frac{3z}{\sqrt{z^{2}+1}}\left ( 1-\frac{z^{2}}{z^{2}+1} \right )$

 

$P=-3t^{3}+t$ với $\frac{z}{\sqrt{z^{2}+1}}=t\in (0;1)$

 

Khảo sát hàm số trên ta thấy $maxP=\frac{2}{9}\Leftrightarrow x=\frac{\sqrt{2}}{2};y=\sqrt{2},z=\frac{\sqrt{2}}{4}$

 

 

 


Bài 17:(Toc Ngan)Cho $a,b,c >0$ và $a+b+c=3$

Chứng minh rằng : $8(\frac{1}{a}+\frac{1}{b}+\frac{1}{c})+9 \geq 10(a^2+b^2+c^2)$

 Bài giải:(babystudymaths)

 

Cách 1

Giải như sau:

Giả sử a là số lớn nhất trong 3 số a,b,c ,thế thì c nhỏ hơn 3 và không nhỏ hơn 1

ta thấy $9=(42a-48)+(42b-\frac{69}{2})+(42c-\frac{69}{2})$

Thay và BĐT ban đầu ta thấy tương đương

$(\frac{8}{b}-10b^{2}+42b-\frac{69}{2})+(\frac{8}{c}-10c^{2}+42c-\frac{69}{2})\geq 10a^{2}-\frac{8}{a}-42a+48\Leftrightarrow \frac{(16-5b)(2b-1)^{2}}{b}+\frac{(16-52)(2c-1)^{2}}{c}\geq \frac{4(5a-1)(a-2)^{2}}{a}$

Áp dụng BCS ,ta có:

VT $\geq \frac{(2b-1+2c-1)^{2}}{\frac{b}{16-5b}+\frac{c}{16-5c}}= \frac{4(a-2)^{2}}{\frac{b}{16-5b}+\frac{c}{16-5c}}$

Lúc này ta chỉ cần chứng minh 

$\frac{a}{5a-1}\geq \frac{b}{16-5b}+\frac{c}{16-5c}$

Mà $\frac{b}{16-5b}+\frac{c}{16-5c}\leq \frac{b}{16-5a}+\frac{c}{16-5a}= \frac{3-a}{16-5a}\leq \frac{a}{5a-1}\Leftrightarrow \frac{1}{(5a-1)(16-5a)}> 0$

ĐÚng theo giả thiết,từ đây ta suy ra đ.p.c.m

Đẳng thức xảy ra khi và chỉ khi a=b=1/2 ,c=2 cùng hoán vị

 

Cách 2: 

Ta có BĐT tương đương

f(abc,a+b+c,ab+bc+ca) =$8.\frac{ab+bc+ca}{abc}+9-10((a+b+c)^{2}-2(ab+bc+ca))\geq 0$

Nhận thấy đây là hàm đơn điệu trên R theo abc nên theo định lý ABC, hàm số đạt cực tiểu khi có 2 biến = nhau, nên a=b=$\frac{3-c}{2}$

Thay vào và chứng minh BĐT 1 biến c ,bài toán trở nên quá đơn giản 

 

 

 

 


Bài 18:(caybutbixanh)Cho $x;y;z> 0$.Chứng minh rằng :

$P=\frac{2xy}{(z+x)(z+y)}+\frac{2yz}{(x+y)(x+z)}+\frac{3xz}{(y+z)(y+x)}\geqslant \frac{5}{3}.$

 

(trích đề thi học sinh giỏi lớp 11-Quảng Bình 2011)

--------------------------------

 

 

 

(T M) Hướng giải:

 

Bằng khai triển trực tiếp ta đưa bất đẳng thức cần chứng minh thành

 

$$xy(x+y)+yz(y+z)+4xz\left ( x+z \right )\geq 10xyz$$

 

Điều này tương đương với

 

$$\frac{x+y}{z}+\frac{y+z}{x}+\frac{4(x+z)}{y}\geq 10$$

 

Áp dụng $AM-GM$ từng cặp là ra.

 

 


Bài 19:(supermath98)Cho các sô dương $a;b;c$ thỏa mãn $\large ab+ac+bc=3abc$. Tìm GTNN của biểu thức: 

 

$\large M=\frac{2\left ( a^{2}b^{2}+b^{2}c^{2}+a^{2}c^{2} \right )+abc}{a^{2}b^{2}c^{2}}$

 

 

Bài giải:(thanhdok14)

 

 

Vì $a, b, c>0$ nên từ điều kiện ban đầu, ta suy ra:

$\frac{1}{a}+\frac{1}{b}+\frac{1}{c}=3$

Đặt: $\left(\frac{1}{a};\frac{1}{b};\frac{1}{c}\right)\to (x;y;z)$

$\Rightarrow x+y+z=3$

$\Rightarrow xy+yz+zx\le 3$

Mặt khác: $M$ được viết lại thành:

$M=2\left(\frac{1}{a^2}+\frac{1}{b^2}+\frac{1}{c^2}\right)+\frac{1}{abc}$

$=2(x^2+y^2+z^2)+xyz$

Lại có: $x^2+y^2+z^2=9-2(xy+yz+zx)$

$xyz\ge \frac{(x+y+z)[4(xy+yz+zx)-(x+y+z)^2]}{9}=\frac{4(xy+yz+zx)-9}{3}$   (theo $schur$)

Từ đó ta có:

$M\ge \frac{4}{3}(xy+yz+zx)-4(xy+yz+zx)+15=\frac{-8}{3}(xy+yz+zx)+15\ge 7$   (vì $xy+yz+zx\le 3$)

Vậy $min M=7\Leftrightarrow a=b=c=1$

 

 


Bài 20:(duaconcuachua)

Cho $a,b,c$ là các số thực dương thỏa mãn $ab+bc+ca=abc$.

Chứng minh rằng $\frac{a^{4}+b^{4}}{ab(a^{3}+b^{3})}+\frac{b^{4}+c^{4}}{bc(b^{3}+c^{3})}+\frac{c^{4}+a^{4}}{ca(c^{3}+a^{3})}\geq 1$

 Bài giải:(Sagittius912)Theo bđt Chebyshev ta có

 

 

$\frac{a^4+b^4}{a^3+b^3}\ge \frac{a+b}{2}$

do đó

 

$\frac{a^{4}+b^{4}}{ab(a^{3}+b^{3})}+\frac{b^{4}+c^{4}}{bc(b^{3}+c^{3})}+\frac{c^{4}+a^{4}}{ca(c^{3}+a^{3})}\ge \frac{a+b}{2ab}+\frac{b+c}{2bc}+\frac{c+a}{2ca}=\frac{ab+bc+ca}{abc}=1$

 

Dấu đẳng thức xảy ra khi $a=b=c=3$

 

 


 

 

 

ở chỗ bài 6 làm sao mk ra dòng thứ 4 trên xuống thế ạ